will mark brainliest!!

picture included ^^^^

please and THANK YOU


^^!!

Answers

Answer 1

Answer:

shift the graph up 1 unit

Step-by-step explanation:

d is the vertical shift

+1 means we shift the graph up 1 unit


Related Questions

Please help explanation if possible

Answers

Answer:

Step-by-step explanation:

Subtract 3x from both sides

3x - 3x - 2y = -3x + y

Combine

-2y = - 3x + 7  

Divide by - 2

-2y/-2 = -3x/-2 + 7/-2

y = 3x/2 - 7/2

(PICTURE) Im really struggling with questions like these at the moment, if you could please help me out thank you

Answers

Answer:

Arthur should select the Mount Joy Pool when there are less than 10 people at the party

Step-by-step explanation:

The initial fee to rent the Woodbridge Pool = $50

The additional fee per person after renting = $5

The initial fee to rent at Mount Joy Pool = 0 (no initial fee)

The fee per person at Mount Joy Pool = $10

The equation of a straight line is y = m·x + c

Ley y  represent the total cost of renting the pool, x, represent the number of persons in the pool, m represent the fee per person and c represent the initial charges, we get;

The total cost of renting Woodbridge Pool, y = 5·x + 50

The total cost of renting the Joy Pool, y = 10·x

Equating both values of y gives;

5·x + 50 = 10·x

∴ 5·x = 50

x = 50/5 = 10

x = 10

From the above equations, the cost of renting the pool is lower for the Joy Pool when there are less than 10 persons at the pool

It will cost the same amount to rent the pool when the number of persons in the pool are 10 persons and the cost of renting the Mount Joy Pool  will be more than the cost of renting the Woodbridge Pool, when there are more than 10 persons

Therefore, Arthur should select the Mount Joy Pool when there are less than 10 people at the party.

A bus has velocity 20m/s towards east and another bus has velocity 15m/s in West direction.If they start to move from a point simultaneously.What distances do they covered in 2 minutes.​

Answers

Answer:

4,200 m or 4.2 km.

First bus: 2400 m

2nd bus: 1800 m.

Step-by-step explanation:

Their combines speed is 20+15 = 35m/s.

Distance = speed * time

2 minutes = 120 seconds so

D = 35 * 120

= 4200 m.

Splitting the distances:

The first bus covers 20 * 120 = 2400m

The second bus covers 15*120 = 1800m.

Answer:

4200m

Step-by-step explanation:

If two objects A and B are moving in the opposite direction with speed xm/s and ym/s respectively,then;

Relative speed =(x+y)m/s

: Relative speed = 20+15=35m/s

time =2min=160sec

DISTANCE=SPEED ×TIME

D=35×120=4200

4200m

Which inequality is represented by the graph?

Answers

Answer:

It will be -5≤×≤4

hi, please solve these three questions for me, i have to shoe solving steps.​

Answers

question 3

Step-by-step explanation:

i only able to show you the step of question 3..so sorry

From an 80ft building the angle of elevation of the top of a taller building is 59 degrees and the angle of depression of the base of this building is 65 degrees determine the height of the taller building

Answers

Answer:

First we find the distance between the two buildings =

Step-by-step explanation:

[tex] \frac{80}{ \tan(65) } = 37.3 ft[/tex]

then the upper part of the taller building =

[tex]37.3 \times \tan(59) = 62 \: ft[/tex]

Now the total height of the taller building

[tex]80 + 62 = 142 \: ft[/tex]

help please I'm doing some math homework ​

Answers

Answer:

Slope is 5/2

Step-by-step explanation:

[tex]{ \boxed{ \bf{slope = \frac{y _{2} - y _{1} }{x _{2} - x _{1}} }}}[/tex]

Substitute the terms:

[tex]{ \tt{slope = \frac{7 - ( - 3)}{4 - 0} }} \\ = \frac{5}{2} [/tex]

Five consecutive multiples of 7 have a sum of 350. What is the smallest of these numbers?
A. 70
B. 56
C. 77
D. 84

Answers

Answer:

B. 56

Step-by-step explanation:

x + (x + 7 ) + ( x + 14 ) + ( x + 21 ) + ( x + 28 ) = 350

( x + x + x + x + x ) + ( 7 + 14 + 21 + 28 ) = 350

5x + 70 = 350

     - 70    - 70

_____________

5x = 280

x = 56

Hope this helps!

The smallest number is 56, the correct option is B.

What is Sum?

The sum is the output of the mathematical operation, Addition.

Let the first number is x, they are multiples of 7,

As they are multiples of 7, the consecutive numbers will be added by 7 for next term.

The 5 consecutive numbers can be written as x, (x+7), (x+14), (x+21), (x+28).

The equation can be formed for the numbers that are given as,

An equation is a mathematical statement that relates an algebraic expression with other expression by an equal sign.

The sum of the multiples is 350

x + (x + 7 ) + ( x + 14 ) + ( x + 21 ) + ( x + 28 ) = 350

Grouping the variables and the constants separately

( x + x + x + x + x ) + ( 7 + 14 + 21 + 28 ) = 350

5x + 70 = 350

Adding (-70) to both sides of the equation

5x = 280

Dividing both sides by 5

x = 56

The value of the first number of the series is obtained.

The first value is the smallest number of the series.

To know more about Sum

https://brainly.com/question/13013054

#SPJ2

Write the equation in slope-intercept form. y = 6(x + 2) + 5x

Answers

Answer:

y=11x+12

Step-by-step explanation:

y = 6(x + 2) + 5x

y=6x+12+5x

y=11x+12

in slope interception form=  y=mx+c

                                                y=11x+12

Y=11x + 12 my brother

Will mark brainlest helppppppp​

Answers

Answer:

6

Step-by-step explanation:

again ?

7 = (3x - 4)/2

14 = 3x - 4

18 = 3x

x = 6

A train travels a distance of 60 km at uniform speed. If the speed of the train was reduced by 10 kmh-1, the time taken to travel the 60km will increase by 1/2h. Find the speed of the train at the beginning.

Answers

Answer:

Initial speed is 32 m/s

At uniform speed, acceleration is 0, (a = 0).

When speed reduced, (v - u) = 2.78 ms-¹, t = 1800 sec, s = 60 ,000 metres.

From first equation of motion:

[tex]{ \boxed{ \bf{v = u + at}}} \\ { \tt{(v - u) = at}}[/tex]

substitute:

[tex]{ \tt{2.78 = (a \times 1800)}} \\ { \tt{acceleration = 0.0015 \: {ms}^{ - 2} }}[/tex]

from second equation of motion:

[tex]{ \boxed{ \bf{s = ut + \frac{1}{2} a {t}^{2} }}}[/tex]

substitute:

[tex]{ \tt{60000 = 1800u + ( \frac{1}{2} \times 0.0015 \times {1800}^{2}) }} \\ { \tt{1800u = 57570}} \\ { \tt{u = 32 \: m {s}^{ - 1} }}[/tex]

Math models helpppp plss if you know about math models answer this pls

Answers

Answer:

A mathematical model is a description of a system using mathematical concepts and language. The process of developing a mathematical model is termed mathematical modeling. Mathematical models are used in the natural sciences and engineering disciplines, as well as in non-physical systems such as the social sciences.

Jeanette ice cream shop sold 10 sundaes with nuts and 7 sundaes without nuts to the total number of sundaes

Answers

The answer to the question is 17
There are 10 sundaes with nuts, and 7 without nuts. Our task is to find the total number of sundaes. We can find this by adding 10 + 7. This gives us 17 sundaes.

I suck at math ik but i need anyones help please

Answers

Answer:

f(2)= 1

f-¹(1)= 2

f-¹(f(2))= 2

I hope I helped you^_^

To wash a window that is 4 meters off the ground, Rafi leans a 5-meter ladder against the side of the building. To reach the window, how far away from the building should Rafi place the base of the ladder?

Answers

Answer:

Base of the ladder is 3 meters away from the building.

Step-by-step explanation:

Let's use Pythagoras theorem to solve.

Pythagoras theorem says,

[tex]a^{2} +b^{2} =c^{2}[/tex]

Here let horizontal distance is "a''

Vertical distance of window is 4 m

So, b=4

The Rafi leans 5 m ladder against the wall. So, c=5.

[tex]a^{2} +4^{2} =5^{2}[/tex]

Simplify it

[tex]a^{2} +16=25[/tex]

Subtract both sides 16

[tex]a^{2} =9[/tex]

Take square root on both sides

a=±3

So, base of the ladder is 3 meters away from the building.

Nina is making pink candles to use as decorations on Valentine's Day. She melts red and white wax together and pours them into a heart-shaped mould. Then, she melts double the amount of red wax and double the amount of white wax together and pours them into a flower-shaped mould. Which candle is a lighter shade of pink?

Answers

Answer:

answer is neither, they are both the same shade.

Step-by-step explanation:

if she doubled the colors exactly then it wouldn't be any different. Also IXL told me I'm right :)

Answer:

Neither.

Step-by-step explanation:

Originally, she used the same amount. For the second one, she used double the same amount, but it was still the amount of red wax as of white wax. So, they both had the same amount, which means it is neither.

Please help !!!!!!!!!!!

Answers

Answer:

98°+62°+6x = 180

160+6x = 180

6x = 180 - 160

6x = 20

x= 20/6

x= 3.33

therefor your answer is 3.33

Find the missing value in each figure below. What does “y” equal?

Answers

Answer:

Step-by-step explanation:

The perpendicular is equal to 6. That's because the left triangle's missing angle is 180 - 45 -90 = 45

The angle in the right triangle is given as 52.

The cos(52) = adjacent side (which we just found to be 6) / y

Multiply both sides by y

y cos(52) = 6

cos(52) = 0.6157

Divide by sides by cos(52)

y = 6 / cos(52)

y = 6 / 0.6157

y = 9.76

find the measure of the indicated angle to the nearest degree​

Answers

[tex]\boxed{\sf sin\Theta=\dfrac{P}{H}}[/tex]

[tex]\\ \sf\longmapsto sin\Theta=\dfrac{16}{26}[/tex]

[tex]\\ \sf\longmapsto sin\Theta=\dfrac{8}{13}[/tex]

[tex]\\ \sf\longmapsto sin\Theta=0.5[/tex]

Convert to p/q form

[tex]\\ \sf\longmapsto sin\Theta=\dfrac{5}{10}[/tex]

[tex]\\ \sf\longmapsto sin\Theta=\dfrac{1}{2}[/tex]

[tex]\\ \sf\longmapsto sin\Theta=sin30[/tex]

[tex]\\ \sf\longmapsto \Theta\approx30°[/tex]

if the radius of a sphere is7 cm find its volume

Answers

Answer:

1437.33 cm square

Step-by-step explanation:

Now we will use the frmula to et the volume of a sphere

volume of sphere is

4÷3pi r cube

where r is the radius of the sphere

find a number such that when 3/4 of it is added to 3½the sum is the same as when 2/3 of it is subtracted from 6½. PLEASE HELP​

Answers

Answer:

- 120

Step-by-step explanation:

call x is the number you want to find

(3/4 )x + 3[tex]\frac{1}{2}[/tex] = (2/3)x - 6[tex]\frac{1}{2}[/tex](3/4)x + (7/2) = (2/3)x - (13/2)(3/4)x - (2/3)x = - (13 /2) - (7/2)(1/12)x = -10x = -10 / (1/12) = -120

cho tam giác ABC vuông tại A < góc B=a chứng minh: a) 1+[tex]tan^{2}[/tex]a=[tex]\frac{1}{sin^{2}a }[/tex]
làm giúp mình với

Answers

LHS:-

[tex]\\ \sf\longmapsto 1+tan^2A[/tex]

[tex]\boxed{\sf tanA=\dfrac{sinA}{cosA}}[/tex]

[tex]\\ \sf\longmapsto 1+\dfrac{sin^2A}{cos^2A}[/tex]

[tex]\\ \sf\longmapsto \dfrac{cos^2A+sin^2A}{cos^2A}[/tex]

[tex]\boxed{\sf cos^2A+sin^2A=1}[/tex]

[tex]\\ \sf\longmapsto \dfrac{1}{cos^2A}[/tex]

[tex]\\ \sf\longmapsto \dfrac{1}{1-sin^2A}[/tex]

[tex]\\ \sf\longmapsto \dfrac{1}{1}-\dfrac{1}{sin^2A}[/tex]

[tex]\\ \sf\longmapsto \dfrac{1}{sin^2A}[/tex]

Hence verified

A team of 5people to be selected from 7women & 6men. Find the number of different teams that could be selected if there must be more women than men in the team

Answers

Answer:i would say 2 differentt teams where there is more then women than men in the team

Step-by-step explanation:

well how i came to this answer is that the team is limited to only 5 people giving the only team where there is more women over men is this first team would be 4 women and 1 men , second team would be 3 women and 2 men anything lower then 3 make its where the team has more men than women so the only options would be 2 team where either they go with 4 women or 3 women and you can't go with more then 4 because then there would be no men in the team which is what the question asks for please go with 2 teams

if this helps please make me brainlist ?!

what is a possible solution to the inequality?

1/4a +1 > 9

Answers

Answer:

a > 32 is one possible solution to the inequality 1/4a +1 > 9.

Step-by-step explanation:

A motorist drove from town P to town Q, a distance of 80 km, in 30 minutes . What is his average speed?​

Answers

Answer:

40km per hour

Step-by-step explanation:

30 minutes × 2 = 60 minutes/1 hour

80 kilometers ÷ 2 = 40 kilometers

40:60


Find the area of triangle ABC.
[PLEASE HELP ME!!]
A.12.16units
B.18.52units
C.31.27units
D.15.14units

Answers

Answer:

D

Step-by-step explanation:

The area (A) of the triangle is calculated as

A = [tex]\frac{1}{2}[/tex] × product of 2 sides × sine ( angle between them ) , that is

A = 0.5 × 5.04 × 6.82 × sin61.73° ≈ 15.14 ( to 2 dec. places )

help me with math pls;(

Answers

Answer:

as it is cbe the volume of cube is l*l*l

Step-by-step explanation:

the answercis 5*5*5= 125

Answer: 125 cm^3

please mark brainliest

Samia created the following tables of values for a linear system. She concluded that there is no
solution to the system

Answers

Since the two linear relations are parallel to each other, no solution to the system exists. Hence, Samia's conclusion is correct.

How many solutions do a system of linear lines have?

A system of equations, involving two linear lines has solutions as follows:

Unique Solution: When the lines intersect, the point of intersection is the solution.No Solution: When the lines are parallel, no solution exists.Infinite Solution: When the lines coincide, then all points on the line become solutions, giving an infinite number of solutions.

How to solve the question?

In the question, we are informed that Samia created the given tables for a linear system, and concluded that no solution exists for the system.

We are asked to comment on her conclusion.

To check for her conclusion, we calculate the slopes of both the lines to check whether the relations are parallel or not, as, for parallel relations, the slopes are equal.

Slope can be calculated using the formula, m = (y₂ - y₁)/(x₂ - x₁), when (x₁, y₁), and (x₂, y₂) are the points on the line.

Thus, the slope for:-

Relation 1, is m₁ = (22 - 8)/(4 - (-3)) = 14/7 = 2.

Relation 2, is m₂ = (12 - (-2))/(4 - (-3)) = 14/7 = 2.

Since the slope of the two relations is equal, that is, m₁ = m₂, and they are not coinciding with each other, we can say that the two relations are parallel to each other.

Since the two linear relations are parallel to each other, no solution to the system exists. Hence, Samia's conclusion is correct.

Learn more about the system of equations at

https://brainly.com/question/17149879

#SPJ2

Please help, need to find f^-1 (x)

Answers

The inverse function [tex]f^{-1}(x)[/tex] is such that

[tex]f\left(f^{-1}(x)\right) = x[/tex]

Plugging [tex]f^{-1}(x)[/tex] into [tex]f(x)[/tex] gives us

[tex]f\left(f^{-1}(x)\right) = \dfrac{3f^{-1}(x) + 3}{5f^{-1}(x) + 6} = x[/tex]

Solve for [tex]f^{-1}(x)[/tex] :

[tex]\dfrac{3f^{-1}(x) + 3}{5f^{-1}(x) + 6} = x \\\\ 3f^{-1}(x)+3=x\left(5f^{-1}(x)+6\right) \\\\ 3f^{-1}(x) + 3 = 5x f^{-1}(x)+6x \\\\ 5xf^{-1}(x)-3f^{-1}(x) = 3 - 6x \\\\ (5x-3)f^{-1}(x)=3-6x \\\\ \boxed{f^{-1}(x)=\dfrac{3-6x}{5x-3}}[/tex]

Given the equation, 1 + 6k = 25, for some value of k, what is the value of k +7 for the same value
of k?

Answers

Answer:

k + 7 = 11

k = 4

Step-by-step explanation:

1 + 6k = 25

25 - 1 = 6k

24 / 6 = k

k = 4

4 + 7 = 11

If my answer is incorrect, pls correct me!

If you like my answer and explanation, mark me as brainliest!

-Chetan K

Answer:

k = 4

The value of k + 7 = 11

Step-by-step explanation:

1 + 6k = 25

To find the value of k we have to use inverse operations

1 + 6k = 25

1 - 1= 0

25 - 1 = 24

This cancels out the 1 in this equation. 25 becomes 24 now because what we do to one side we do to the other.

6k = 24

To find k we have to divide 6 from 6 and 24

6/6= 0

24/6= 4

Now we are left with the variable k and 4

Therefore, k = 4.

To solve the second part of the equation we plug in k to the equation

k = 4

so the equation becomes 4 + 7

4 + 7 = 11

Other Questions
There is a circle with a center of 0,0 on a coordinate plane. There is one point on the circle's circumference in which the x:y ratio is 3:1. What is a possible coordinate? Blade Breeze Company manufactures ceiling fans and uses an activity-based costing system. Each ceiling fan has 20 separate parts. The direct materials cost is $70, and each ceiling fan requires 2.50 hours of machine time to manufacture. Additional information is as follows: Activity Allocation Base Predetermined Overhead Allocation Rate Materials handling Number of parts $ 0.08 Machining Machine hours 7.20 Assembling Number of parts 0.35 Packaging Number of finished units 2.80What is the cost of machining per ceiling fan? (Round any intermediate calculations and your final answer to the nearest cent.) A) $18.00 B) $70.00 C) $144.00 D) $196.00 IT will be shown as hypertension if the blood pressure of an individual is ______a)120/80b)130/70c)140/90d)100/80 A chef is going to use a mixture two different brands of Italian dressing the first spring and days 5% vinegar the second brain contains 15% vinegar the sheriff wants to make 390$ ml addressing that is 9% vinegar how much of each brand should she use Help help help help help Jeremy is buying a new car. The total cost, including tax, is $18275. If the tax rate is 7.5% , what is the sticker price of the car? 18The length of a rectangle is twice as long as the width of the rectangle.The area of the rectangle is 32 cm.Draw the rectangle on the centimetre grid..454B2%I did it wrong can someone help me Give reason. Mosquitoes and housefly are placed in the phylum arthropoda Which answer below uses atransitional word or phrase?A. I hope we can go to the store later -- we need to keepcleaning the house.B. I hope we can go to the store later; in the meantime, weneed to keep cleaning the house.C. We need to keep cleaning the house, I hope we can go tothe store later.Copyright 2021 Acelus Corporation. All Rights R A pendulum's height is modeled by the function h(t) = 4 cos(pi/4*t) + 8 where h is themeasure of the pendulum's height in feet and t is the number of seconds since themaximum height. How many seconds does it take the pendulum to complete onefull swing? An organization consists of 8,684 employees. They decided to conduct a survey about their new vacation policies. The organization surveyed 884 of their employees and found that 36% of those surveyed disliked the new vacation policies. Assuming a 95% confidence level, which of the following statements holds true? A city filed eminent domain proceedings in order to obtain 40 beach houses fronting a particularly attractive stretch of shoreline. As part of an elaborate plan to increase the city's tourist trade and revive the local economy, the city planned to sell the beach houses to a company that would demolish the houses and build a luxury hotel in their place. The owners of the beach houses have challenged the city's exercise of eminent domain, contending only that the city's plan is unconstitutional. Will the owners of the beach houses be likely to prevail If a fixed asset, such as a computer, were purchased on January 1st for $3,750 with an estimated life of 3 years and a salvage or residual value of $150, the journal entry for monthly expense under straight-line depreciation is: (Note: EOM indicates the last day of each month.) What is the probability of a red on this spinner?Be sure to reduce. Solve the inequality (help pls) oque significa verbal e no verbal During the _____ stage of the general adaptation syndrome, the emotions are aroused and the body prepares its defense systems to respond.a. resistanceb. alarmc. exhaustiond. avoidance Vo u th k XX, Bc K c mt cuc vn ng ci cch vn ha x hi theo li t sn l Living in the northern hemisphere, you decide to plant tulip bulbs in the spring. You are disappointed in the summer when they do not bloom. You had forgotten that tulip bulbs need a period of vernalization in order for them to bloom. What change should you make? a) Plant the tulip bulbs in soil that has an abundance of nutrients, especially phosphorus. b) Make sure that the tulips are kept in shady areas as too much sun will not allow them to sprout c) Be careful to give the tulips plenty of water while they are germinating, but reduce the amount as the plants grow. d) Ensure the plants are established in a region that will receive long periods of dark as this is required for the tulips to bloom. e) Plant the tulip bulbs in the fall as opposed to the spring. The following ordered pairs represent a function: {(-3, 1), (1, -2), (3, 0), (4, 5)}.True or False